• Asignatura: Física
  • Autor: carlitos101976
  • hace 8 años

Un protón se mueve con una velocidad v ⃗= (i ̂+ 2j ̂- k ̂ ) m/s en una región donde el campo magnético tiene un valor B ⃗= (4i ̂- 3j ̂+ (2k) ̂ ) T. ¿Cuál es la magnitud de la fuerza magnética que experimenta esta carga?

Respuestas

Respuesta dada por: Osm867
3

Respuesta.


Para resolver este problema hay que aplicar la siguiente ecuación:


F = q*(v x B)


Datos:


q = 1.602 x 10⁻¹⁹ C

v = (1, 2, -1) m/s

B = (4, -3, 2) T


En primer lugar se calcula v x B.


x = (2*2) - (-1*-3) = 1

y = -(1*2) + (-1*4) = -6

z = (-3*1) - (4*2) = -11


v x B = (1, -6, -11)


Finalmente:


F = (1.602 x 10⁻¹⁹)*(1, -6, -11)

F = (1.602 x 10⁻¹⁹, -9.612 x 10⁻¹⁹, 1.7622 x 10⁻¹⁸) N

Preguntas similares